Question 1 (5 points)
The line segment AB with endpoints A(-3, 6) and B(9, 12) is dilated with a scale
factor 2/3 about the origin. Find the endpoints of the dilated line segment.
OA) (-2, 4), (6,8)
B) (2, 4). (6,8)
OC) (4, -2), (6,8)
OD) (-2,4), (8,6)​

Answers

Answer 1

Answer: A) (-2, 4), (6,8)

Step-by-step explanation:

When a point (x,y) is dilated by a scale factor of k , then the new points is given by (kx,ky).

Given: The line segment AB with endpoints A(-3, 6) and B(9, 12) is dilated with a scale factor [tex]\dfrac23[/tex] about the origin.

Let A' and B' b the endpoints of the dilated line segment.

Then, [tex]A'(\dfrac{2}{3}(-3), \dfrac23(6))=A'(-2,4)[/tex]

[tex]B'(\dfrac{2}{3}(9), \dfrac23(12))=B'(6,8)[/tex]

Hence, the correct option is A) (-2, 4), (6,8)


Related Questions

The force of gravity on an object varies directly with its mass. The constant of variation due to gravity is 32.2 feet per second squared. Which equation represents F, the force on an object due to gravity according to m, the object’s mass? F = 16.1m F = F equals StartFraction 16.1 Over m squared EndFraction. F = 32.2m F = F equals StartFraction 32.2 Over m squared EndFraction.

Answers

Answer:

F = 32.2m

Step-by-step explanation:

According to newton second law, the force of gravity on an object varies directly with its mass and it is expressed mathematically as Fαm i.e

F = mg where;

F is the force of gravity

m is the mass of the body

g is the proportionality constant known as the acceleration due to gravity.

If the constant of variation due to gravity is 32.2ft/s², the equation that represents F, the force on an object due to gravity according to m, the object’s mass can be gotten by substituting g = 32.2 into the formula above according to the law as shown;

F = m*32.2

F =32.2m

Hence the required equation is F = 32.2m

musah stands at the center of a rectangular field . He first takes 50 steps north, then 25 step west and finally 50 steps on a bearing of 315°. How far west and how far north is Musah final point from the center?

Answers

Answer:

85.36 far north from the center

10.36 far east from the center

Step-by-step explanation:

The extra direction taken in the north side is x

X/sin(360-315)=50/sin 90

Sin 90= 1

X/sin 45= 50

X= sin45 *50

X= 0.7071*50

X= 35.355 steps

X= 35.36

Then the west direction traveled

West =√(50² - 35.355²)

West = √(2500-1249.6225)

West= √1250.3775

West= 35.36 steps

But this was taken in an opposite west direction

From the center

He is 35.36 +50

= 85.36 far north from the center

And

25-35.36=-10.36

10.36 far east from the center

You sell tickets at school for fundraisers. You sold car wash tickets, silly string fight tickets and dance tickets – for a total of 380 tickets sold. The car wash tickets were $5 each, the silly sting fight tickets were $3 each and the dance tickets were $10 each. If you sold twice as many silly string tickets as car wash tickets, and you have $1460 total. Write the matrix in the box below. Write the solution set for this system and include any necessary work.

Answers

Answer:

Matrix :

[tex]\begin{bmatrix}1&1&1&|&380\\ 5&3&10&|&1460\\ -2&1&0&|&0\end{bmatrix}[/tex]

Solution Set : { x = 123, y = 246, z = 11 }

Step-by-step explanation:

Let's say that x represents the number of car wash tickets, y represents the number of silly sting fight tickets, and z represents the number of dance tickets. We know that the total tickets = 380, so therefore,

x + y + z = 380,

And the car wash tickets were $5 each, the silly sting fight tickets were $3 each and the dance tickets were $10 each, the total cost being $1460.

5x + 3y + 10z = 1460

The silly string tickets were sold for twice as much as the car wash tickets.

y = 2x

Therefore, if we allign the co - efficients of the following system of equations, we get it's respective matrix.

System of Equations :

[tex]\begin{bmatrix}x+y+z=380\\ 5x+3y+10z=1460\\ y=2x\end{bmatrix}[/tex]

Matrix :

[tex]\begin{bmatrix}1&1&1&|&380\\ 5&3&10&|&1460\\ -2&1&0&|&0\end{bmatrix}[/tex]

Let's reduce this matrix to row - echelon form, receiving the number of car wash tickets, silly sting fight tickets, and dance tickets,

[tex]\begin{bmatrix}5&3&10&1460\\ 1&1&1&380\\ -2&1&0&0\end{bmatrix}[/tex] - Swap Matrix Rows

[tex]\begin{bmatrix}5&3&10&1460\\ 0&\frac{2}{5}&-1&88\\ -2&1&0&0\end{bmatrix}[/tex] - Cancel leading Co - efficient in second row

[tex]\begin{bmatrix}5&3&10&1460\\ 0&\frac{2}{5}&-1&88\\ 0&\frac{11}{5}&4&584\end{bmatrix}[/tex] - Cancel leading Co - efficient in third row

[tex]\begin{bmatrix}5&3&10&1460\\ 0&\frac{11}{5}&4&584\\ 0&\frac{2}{5}&-1&88\end{bmatrix}[/tex] - Swap second and third rows

[tex]\begin{bmatrix}5&3&10&1460\\ 0&\frac{11}{5}&4&584\\ 0&0&-\frac{19}{11}&-\frac{200}{11}\end{bmatrix}[/tex] - Cancel leading co - efficient in row three

And we can continue, canceling the leading co - efficient in each row until this matrix remains,

[tex]\begin{bmatrix}1&0&0&|&\frac{2340}{19}\\ 0&1&0&|&\frac{4680}{19}\\ 0&0&1&|&\frac{200}{19}\end{bmatrix}[/tex]

x = 2340 / 19 = ( About ) 123 car wash tickets sold, y= 4680 / 19 =( About ) 246 silly string fight tickets sold, z = 200 / 19 = ( About ) 11 tickets sold

Which choice is equivalent to the expression below? √-12

A. 12i
B. -12i
C. -2√3
D. 2i √3
E. -2√3i

PLEASE DON’T GUESS

Answers

Answer:

D.  2i√3

Step-by-step explanation:

You have the expression √-12.  You can divide the number in the radical sign into the numbers that make up the expression.  After you do this, you will be able to take numbers out of the radical sign

√(-12)

√(-1 × 4 × 3)

√-1 = i

√4 = 2

√3 = √3

2i√3

The answer is D.

Select the correct answer from each drop-down menu.
Nirja has 24 marbles. The number of marbles Nirja has is 6 more than the number of marbles Tim has.
If Tim has x marbles, the equation that represents the situation is
The value of x that makes the equation true is
Reset
Next

Answers

Answer:

24 = x+6

x = 18

Step-by-step explanation:

N = 24

T = x

N = x+6  

24 = x+6

Subtract 6 from each side

24-6 = x+6-6

18 = x

Time has 6 marbles

Nirja has 6 more than Tim,

So you can subtract 6 from 24 to find x:

24-6 = x

Or you can add 6 to x to equal 24:

x + 6 = 24

You don't list the choices but it should be one of these.

Solve:

24 - 6 = x

x = 18

f(x) = -3x + 7
What is f (0)?

Answers

f(0) = -3(0) + 7
f(0) = 7

Answer:

f(0) = 7

Step-by-step explanation:

f(x) = -3x + 7

Let x =0

f(0) = -3*0 + 7

f(0) = 7

can you please help ?

Answers

Answer:

69

Step-by-step explanation:

The order of operations is PEMDAS; parentheses, exponents, multiplication and division, and finally addition and subtraction.

We know that x is the first row, and if there are 30 spots in the first row, then x=30. Using this information, all we have to do now is plug in 30 for x and solve.

[tex]\frac{5(x)}{2} -6[/tex]

[tex]\frac{5(30)}{2}-6[/tex]

[tex]\frac{150}{2}-6[/tex]

[tex]75-6[/tex]

[tex]69[/tex]

use the product of powers property to simplify the numeric expression.

4 1/3 • 4 1/5 = _____

Answers

Answer:

The value of [tex]4^{\dfrac{1}{3}} {\cdot} 4^{\dfrac{1}{5}[/tex]  is  [tex]4^{\dfrac{8}{15}}[/tex] .

Step-by-step explanation:

We need to simplify the numeric expression using property. The expression is as follows :

[tex]4^{\dfrac{1}{3}} {\cdot} 4^{\dfrac{1}{5}[/tex]

The property to be used is : [tex]x^a{\cdot} x^b=x^{a+b}[/tex]

This property is valid if the base is same. Here, base is x.

In this given problem, x = 4, a = 1/3 and b = 1/5

So,

[tex]4^{\dfrac{1}{3}} {\cdot} 4^{\dfrac{1}{5}}=4^{\dfrac{1}{3}+\dfrac{1}{5}}\\\\=4^{\dfrac{5+3}{15}}\\\\=4^{\dfrac{8}{15}}[/tex]

So, the value of [tex]4^{\dfrac{1}{3}} {\cdot} 4^{\dfrac{1}{5}[/tex]  is  [tex]4^{\dfrac{8}{15}}[/tex] .

please help
-3(-4x+4)=15+3x

Answers

Answer:

x=3

Step-by-step explanation:

● -3 (-4x+4) = 15 + 3x

Multiply -3 by (-4x+4) first

● (-3) × (-4x) + (-3)×(4) = 15 + 3x

● 12 x - 12 = 15 +3x

Add 12 to both sides

● 12x - 12 + 12 = 15 + 3x +12

● 12 x = 27 + 3x

Substract 3x from both sides

● 12x -3x = 27 + 3x - 3x

● 9x = 27

Dividr both sides by 9

● 9x/9 = 27/9

● x = 3

The radius of a sphere is measured as 7 centimeters, with a possible error of 0.025 centimeter.

Required:
a. Use differentials to approximate the possible propagated error, in cm3, in computing the volume of the sphere.
b. Use differentials to approximate the possible propagated error in computing the surface area of the sphere.
c. Approximate the percent errors in parts (a) and (b).

Answers

Answer:

a) dV(s)  =  15,386 cm³

b) dS(s) = 4,396 cm²

c) dV(s)/V(s) = 1,07 %    and   dS(s)/ S(s)  =  0,71 %

   

Step-by-step explanation:

a) The volume of the sphere is

V(s) = (4/3)*π*x³        where x is the radius

Taking derivatives on both sides of the equation we get:

dV(s)/ dr  =  4*π*x²    or

dV(s)  =  4*π*x² *dr

the possible propagated error in cm³ in computing the volume of the sphere is:

dV(s)  = 4*3,14*(7)²*(0,025)

dV(s)  =  15,386 cm³

b) Surface area of the sphere is:

V(s) = (4/3)*π*x³  

dV(s) /dx  =  S(s) = 4*π*x³

And

dS(s) /dx  = 8*π*x

dS(s) = 8*π*x*dx

dS(s) = 8*3,14*7*(0,025)

dS(s) = 4,396 cm²

c) The approximates errors in a and b are:

V(s) =  (4/3)*π*x³     then

V(s) = (4/3)*3,14*(7)³

V(s) = 1436,03 cm³

And  the possible propagated error in volume is from a)  is

dV(s)  =  15,386 cm³

dV(s)/V(s)  = [15,386 cm³/1436,03 cm³]* 100

dV(s)/V(s) = 1,07 %

And for case b)

dS(s) = 4,396 cm²

And the surface area of the sphere is:

S(s) =  4*π*x³        ⇒   S(s) =  4*3,14*(7)²    ⇒ S(s) = 615,44 cm²

dS(s) = 4,396 cm²

dS(s)/ S(s)  =  [ 4,396 cm²/615,44 cm² ] * 100

dS(s)/ S(s)  =  0,71

paul worked 50 hours last week. if he earns $10 per hour plus time-and-a-half for any hours worked beyond 40 in a week, how much did he earn last week?

Answers

Answer: 4150

Step-by-step explanation:

You take the 50, becuse the amount earned increases once you surpass 40 you do 40 x 10 and that = 4000 then you take the remaining 10 and times that by 15 (becuse after 40 it is 1.5 of what you where earning before you hit 40 hours and half of ten is 5 so you do 10 plus 5 and times that by 10) then add both numbers together and you have 4150! Hope that helped!

a vegetable garden and he's around the path of seemed like a square that together are 10 ft wide. The path is 2 feet wide. Find the total area of the vegetable garden and path​

Answers

Answer:

Garden: 36 square feet

Path: 64 square feet

Step-by-step explanation:

Let's first find the total area. The total area will be 100 square feet since the side length is 10. Since the path is 2 feet wide and on all sides, that means that the inside square will have a side length of 6. That means that the vegetable garden is 36 square feet. The path will be 100 - (the garden), and the garden is 36 square feet, which means the outer path will be 64.

6x - 10 = 4(x + 3) x = ? x = 9 x = 10 x = 11 x = 12

Answers

Answer:

x=11

Step-by-step explanation:

Answer:

x = 11

Step-by-step explanation:

6x - 10 = 4(x+3)

6x - 10 = 4*x + 4*3

6x - 10 = 4x + 12

6x - 4x = 12 + 10

2x = 22

x = 22/2

x = 11

check:

6*11 - 10 = 4(11+3)

66 - 10 = 4*14 = 56

A buoy floating in the sea is bobbing in simple harmonic motion with amplitude 13 in and period 0.25 seconds. Its displacement d from sea level at time t=0 seconds is 0in, and initially it moves downward. (Note that downward is the negative direction.)Required:Give the equation modeling the displacement d as a function of time t.

Answers

Answer:

The equation is [tex]x(t) = -13 cos (8 \pi t )[/tex]

Step-by-step explanation:

From the question we are told that

      The  amplitude is  [tex]A = 13 \ in[/tex]

       The period is  [tex]T = 0.25[/tex]

Generally the displacement function for a simple harmonic motion is mathematically represented as

        [tex]x(t) = A cos (wt )[/tex]

Here  [tex]w[/tex] is the angular frequency which is mathematically represented as

          [tex]w = \frac{2 \pi }{T}[/tex]

substituting values

          [tex]w = \frac{2 \pi }{ 0.25}[/tex]

          [tex]w = 8\pi[/tex]

Given that at t =  0  the displacement is equal to 0 it means that there is no phase shift and also  we are told that it is initially moving downward which implies that its Amplitude is  [tex]A = -13\ in[/tex]

So the equation modeling the displacement d as a function of time t is mathematically represented as

          [tex]x(t) = -13 cos (8 \pi t )[/tex]

The coffee cups can hold 7/9 of a pint of liquid. If Emily pours 2/3 of a pint of coffee into a cup,how much milk can a customer add? PLZ HELP!​

Answers

Answer:

1/9

Step-by-step explanation:

easy 2/3 is equivalent to 6/9. So there is 1/9 of a pint left

write 32 1/2 in radical form​

Answers

Answer:

Nothing further, the simplest answer is 32 1/2

Step-by-step explanation:

A researcher wishes to examine the relationship between years of schooling completed and the number of pregnancies in young women. Her research discovers a linear relationship, and the least squares line is: ˆ y = 3 − 5 x y^=3-5x where x is the number of years of schooling completed and y is the number of pregnancies. The slope of the regression line can be interpreted in the following way:
1.) When amount of schooling increases by one year, the number of pregnancies decreases by 4.
2.) When amount of schooling increases by one year, the number of pregnancies increases by 4.
3.) When amount of schooling increases by one year, the number of pregnancies increases by 5.
4.) When amount of schooling increases by one year, the number of pregnancies decreases by 5.

Answers

Answer:

1. When amount of schooling increases by one year, the number of pregnancies will decrease by 4.

Step-by-step explanation:

Regression analysis is a statistical technique which is used for forecasting. It determines the relationship between two variables. It determines the relationship of two or more dependent and independent variables. It is widely used in stats to find trend in the data. It helps to predict the values of dependent and independent variables. In the given question, there is regression equation given. X and Y are considered as dependent variables. When number of schooling increases by 1 year then number of pregnancies will decrease by 4

In a genetics experiment on peas, one sample of offspring contained green peas and yellow peas. Based on those results, estimate the probability of getting an offspring pea that is green. Is the result reasonably close to the value of that was expected? 350 127 3 4 The probability of getting a green pea is approximately . (Type an integer or decimal rounded to three decimal places as needed.) Is this probability reasonably close to ? Choose the correct answer below. 3 4 A. No, it is not reasonably close. B. Yes, it is reasonably close.

Answers

Complete Question

In a genetic experiment on peas, one sample of offspring contained 436 green peas and 171 yellow peas. Based on those results, estimate the probability of getting an offspring pea that is green. Is the result reasonably close to the value of 3/4 that was expected? The probability of getting a green pea is approximately: Is the probability reasonably close to 3/4?

Answer:

The  probability is  [tex]P(g) =0.72[/tex]

Yes the result is reasonably close

Step-by-step explanation:

From the question we are told that

   The  number of  of  green peas is  [tex]g = 436[/tex]

     The number of yellow peas is  [tex]y = 171[/tex]

   The sample size is  [tex]n = 171 + 436 = 607[/tex]

The probability of getting an offspring pea that is green is mathematically represented as

       [tex]P(g) = \frac{g}{n}[/tex]

        [tex]P(g) = \frac{436}{607}[/tex]

        [tex]P(g) =0.72[/tex]

Comparing  [tex]P(g) =0.72[/tex]  to   [tex]\frac{3}{4} = 0.75[/tex] we see that the result is reasonably close

what are the next terms in the number pattern -11, -8, -5, -2, 1

Answers

Answer:

4, 7, 10, 13

Step-by-step explanation:

Hey there!

Well in the given pattern,

-11, -8, -5, -2, 1

we can conclude that the pattern is +3 every time.

-11 + 3 = -8

-8 + 3 = -5

-5 + 3 = -2

-2 + 3 = 1

And so on

4, 7, 10, 13

Hope this helps :)

Please answer this correctly without making mistakes

Answers

Answer:

151 9/19

Step-by-step explanation:

Step-by-step explanation:

Option A is the correct answer because it is equal to 151.47

The range of values for x?

Answers

Answer:

x = 32

but

I would say anything from 30 to 33

but truly i have no clue about the range

Step-by-step explanation:

3x-9=87 (because 180 -93 =87)

3x = 96

x = 32

Answer:

it is 32

Step-by-step explanation:

If the average fixed cost (AFC) of producing 5 bags of rice is $20.00, the average fixed cost of producing 10 bags will be​

Answers

Answer:$40.00

Step-by-step explanation:first divide 20 by 5 and the answer will be 4. now multiply 10 into 4 and you'll get the answer $40.00

6(x + 2) = 30Solve the following linear equation

Answers

Answer:

[tex]\huge \boxed{x=3}[/tex]

Step-by-step explanation:

[tex]6(x+2)=30[/tex]

[tex]\sf Divide \ both \ sides \ by \ 6.[/tex]

[tex]x+2=5[/tex]

[tex]\sf Subtract \ 2 \ from \ both \ sides.[/tex]

[tex]x=3[/tex]

Answer:

3

Step-by-step explanation:

30 = 6(x+2)

30/6 = 5

5 = x+2

5-2 = 3

3=x

This is a pretty simple question and I tried to make it as simple as possible when explaining it.

Find the first term in the sequence when u(subscript)31=197 and d= 10.

Answers

Answer:

197 = 10(31-1) + a

197 = 300 + a

-103 = a

Determine the value(s) for which the rational expression 2x^2/6x is undefined. If there's more than one value, list them separated by a comma, e.g. x=2,3.

Answers

Answer:

0

Step-by-step explanation:

Hello, dividing by 0 is not defined. so

[tex]\dfrac{2x^2}{6x}[/tex]

is defined for x different from 0

This being said, we can simplify by 2x

[tex]\dfrac{2x^2}{6x}=\dfrac{2x*x}{3*2x}=\dfrac{1}{3}x[/tex]

and this last expression is defined for any real number x.

Thank you

Luke owns a trucking company. For every truck that goes out, Luke must pay the driver $17 per hour of driving and also has an expense of $1.75 per mile driven for gas and maintenance. On one particular day, the driver drove an average of 40 miles per hour and Luke's total expenses for the driver, gas and truck maintenance were $522. Write a system of equations that could be used to determine the number of hours the driver worked and the number of miles the truck drove. Define the variables that you use to write the system.

Answers

Answer:

17h+1.75m=522 m=40h

Step-by-step explanation:

Let h=  {the number of hours the driver drove}

Let m=  the number of miles driven

The driver makes $17 for each hour working, so if the driver worked for hh hours, Luke would have to pay him 17h17h dollars. The cost of gas and maintenance is $1.75 per mile, so for mm miles Luke's costs would be 1.75m1.75m dollars. The total cost of the route 17h+1.75m17h+1.75m equals \$522:$522:

17h+1.75m=522

17h+1.75m=522

Since the driver drove an avearge of 40 miles per hour, if the driver drove  hour, he would have driven 40 miles, and if the driver drove hh hours, he would have driven 40h40h miles, therefore mm equals 40h:40h:

m=40h

m=40h

Write System of Equations:

​  

 

17h+1.75m= 522

m=40h

The truck is going for a run for 6 hours and the system of the equation to solve a further problem related to this is [tex]\rm{Cost}=17x+1.75y[/tex]

The following are the different costs of the truck that Luke must be pay while running a truck:

Luke must pay the driver $17 per hour of driving.A truck has an expense of $1.75 per mile driven for gas and maintenance.

Let ' x ' be the total time of driving a truck in hours.

and ' y ' be the total mile distance that is covered by the truck.

Therefore, the system of the equation for the overall running cost for a truck is given below.

[tex]\rm{Cost}=17x+1.75y[/tex]

Now, On one particular day, the driver drove an average of 40 miles per hour, and Luke's total expenses for the driver, gas and truck maintenance were $522.

Thus,

The total distance traveled by truck is 40x.

That is,

[tex]y=40x[/tex]

Substitute the values and solve them further.

[tex]522=17x+1.75y\\522=17x+1.75 \times 40x\\522=17x+70x\\522=87x\\x=6[/tex]

Thus, the truck is going for a run for 6 hours and the system of the equation to solve the further problems related to this is [tex]\rm{Cost}=17x+1.75y[/tex]

To know more about variables, please refer to the link:

https://brainly.com/question/14393109

The cost of performance tickets and beverages for a family of four can be modeled using the equation 4x+12=48,where x represents the cost of a. Ticket.how much is one ticket

Answers

Answer:

x=9; one ticket is $9

Step-by-step explanation:

4x+12=48

4x=48-12

4x=36

x=36/4

x=9

A spinner has 10 equally sized sections, 5 of which are gray and 5 of which are blue. The spinner is spun twice. What is the probability that the first spin lands on gray and the second spin lands on blue? Write your answer as a fraction in the simplest form.

Answers

Answer:

[tex]P(Gray\ and\ Blue) = \frac{1}{4}[/tex]

Step-by-step explanation:

Given

[tex]Sections = 10[/tex]

[tex]n(Gray) = 5[/tex]

[tex]n(Blue) = 5[/tex]

Required

Determine P(Gray and Blue)

Using probability formula;

[tex]P(Gray\ and\ Blue) = P(Gray) * P(Blue)[/tex]

Calculating P(Gray)

[tex]P(Gray) = \frac{n(Gray)}{Sections}[/tex]

[tex]P(Gray) = \frac{5}{10}[/tex]

[tex]P(Gray) = \frac{1}{2}[/tex]

Calculating P(Gray)

[tex]P(Blue) = \frac{n(Blue)}{Sections}[/tex]

[tex]P(Blue) = \frac{5}{10}[/tex]

[tex]P(Blue) = \frac{1}{2}[/tex]

Substitute these values on the given formula

[tex]P(Gray\ and\ Blue) = P(Gray) * P(Blue)[/tex]

[tex]P(Gray\ and\ Blue) = \frac{1}{2} * \frac{1}{2}[/tex]

[tex]P(Gray\ and\ Blue) = \frac{1}{4}[/tex]

A package of 8-count AA batteries costs $6.40. A package of 20-count AA batteries costs $15.80. Which statement about the unit prices is true?

Answers

Answer:

The unit price of the 20 pack is $0.79 and the unit price for the 8 pack is $0.80.

Step-by-step explanation:

Simply Take the price of the pack of batteries divided by the number within the pack.

$6.40 / 8 == $0.80

$15.80 / 20 == $0.79

Cheers.

The question is incomplete. You can find the missing content below.

A package of 8-count AA batteries costs $6.40. A package of 20-count Of batteries costs $15.80. Which statement about the unit prices is true?

A) The 8-count pack of AA batteries has a lower unit price of $0.79 per battery.

B) The 20-count pack of AA batteries has a lower unit price of $0.80 per battery.

C) The 8-count pack of AA batteries has a lower unit prices of $0.80 per battery.

D) The 20-count pack of AA batteries has a lower unit price of $0.79 per battery.

The correct option is Option D: The 20-count pack of AA batteries has the lower price of $0.79 per battery.

What is inequality?

Inequality is the relation between two numbers or variables or expressions showing relationships like greater than, greater than equals to, lesser than equals to, lesser than, etc.

For example 2<9

A package of 8-count AA batteries has cost = $6.40.

cost per unit count AA batteries will be= total cost of AA batteries/ number of AA batteries

= $6.40/8= $0.8

A package of 20-count AA batteries has cost = $15.80.

cost per unit count AA batteries will be= total cost of AA batteries/ number of AA batteries

= $15.80/20= $0.79

As 0.79<0.8

cost of 20-count AA batteries <  cost of 8-count AA batteries

Therefore the correct option is Option D: The 20-count pack of AA batteries has the lower price of $0.79 per battery.

Learn more about inequality

here: https://brainly.com/question/11613554

#SPJ2

The chief business officer of a construction equipment company arranges a loan of $9,300, at 12 1 /8 % interest for 37.5 months. Find the amount of interest. (Round to the nearest cent)

a. $2,761.21


b. $3,583.83


c. $3,523.83


d. $3,722.47

Answers

Answer:

C). $3523.83

Step-by-step explanation:

loan of principles p= $9,300,

at rate R= 12 1 /8 % interest

Rate R = 12.125%

for duration year T = 37.5 months

T= 37.5/12 = 3.125 years

Interest I=PRT/100

Interest I =( 9300*12.125*3.125)/100

Interest I = (352382.8125)/100

Interest I = 3523.83

Interest I= $3523.83

Other Questions
Following is information on two alternative investments being considered by Jolee Company. The company requires a 6% return from its investments. (PV of $1, FV of $1, PVA of $1, and FVA of $1). (Use appropriate factor(s) from the tables provided.) Project A Project B Initial investment $ (174,325 ) $ (152,960 ) Expected net cash flows in year: 1 41,000 44,000 2 60,000 53,000 3 72,295 68,000 4 87,400 81,000 5 59,000 30,000For each alternative project compute the net present value. Solve systems of equations 15 points NOT CLICKBAIT!!! -6y+11y= -36 -4y+7x= -24 RNA polymerase is an enzyme that copies (1 point)o DNA into DNA.O RNA into mRNAO mRNA into tRNA.DNA into RNA Read the following speech excerpt and then select the correct answer to the question below: President George W. Bushs speech to the troops on the USS Abraham Lincoln Our mission continues. Al-Qaida is wounded, not destroyed. The scattered cells of the terrorist network still operate in many nations, and we know from daily intelligence that they continue to plot against free people. The proliferation of deadly weapons remains a serious danger. The enemies of freedom are not idle, and neither are we. Our government has taken unprecedented measures to defend the homeland and we will continue to hunt down the enemy before he can strike. The war on terror is not over, yet it is not endless. We do not know the day of final victory, but we have seen the turning of the tide. No act of the terrorists will change our purpose, or weaken our resolve, or alter their fate. Their cause is lost. Free nations will press on to victory. Other nations in history have fought in foreign lands and remained to occupy and exploit. Americans, following a battle, want nothing more than to return home. And that is your direction tonight. After service in the Afghan and Iraqi theaters of war after 100,000 miles, on the longest carrier deployment in recent history you are homeward bound. Some of you will see new family members for the first time 150 babies were born while their fathers were on the Lincoln. Your families are proud of you, and your nation will welcome you. How does the sentence "Free nations will press on to victory" relate to the central idea of the speech? It furthers the idea that service people returning home will be called back out quite soon. It supports the main point that this is an ongoing fight that America will not abandon. It challenges the notion that Afghan people need an American presence to ensure their safety. It adds to the idea that it is essential to remove any American military presence beyond in other lands. Which statement describes one feature of Rutherford's model of the atom?O The atom is mostly empty space.O The atom cannot be divided into smaller particles.O Electron clouds are regions where electrons are likely to be found.O The electrons are located within the positive material of the nucleus. this one from maths pls help which category would a person who has an IQ of 84 belong ? In a circle, an arc measuring 130 is what percentage of the circumference of the circle The following are scores obtained by some students in a test. 8 18 10 14 18 11 13 14 13 17 15 8 16 13. Find the mode of the distribution Read the sentence and choose the option with the correct verb in the future tense. El prximo mes, Fabio ________ sobre el ciclo de la vida en la clase de ciencias. hablar hablaremos har heremos Ford Motor Company is discussing new ways to recapitalize the firm and raise additional capital. Its current capital structure has a % weight in equity, % in preferred stock, and % in debt. The cost of equity capital is %, the cost of preferred stock is %, and the pretax cost of debt is %. What is the weighted average cost of capital for Ford if its marginal tax rate is %? A________ is a plan in which an individual balances available resources and expenses. The place you get your hair cut has two nearly parallel mirrors 6.5 m apart. As you sit in the chair, your head is The firm has total fixed costs of $9 and a constant marginal cost of $3 per unit. The firm will maximize profit with a. 9 units of output. b. 15 units of output. c. 21 units of output. d. 30 units of output. I need helps will give you a good rating. Based upon what you have learned in this class, write a psychological explanation forwhat is going on in this scenario; and suggestions to help Ella improve her life.Ella is a 25 year old woman, married with two children. Her daughters are ages 1 and3. She is a university graduate and her partner brings in a good income. Over thepast six months Ella has felt tired, lost weight, and does not seem to find enjoymentin activities that she usually enjoyed.Today she awoke, sat on the couch and watched her two daughters play in the livingroom for the morning. She changed the diapers of her youngest daughter (Nora) andsent her oldest daughter (Addy) to the bathroom. After feeding them lunch, shelooked out the window. She watched as an older man fell on the side-walk, andstruggled for 10 minutes to get back on his feet. Eventually Ella lay down on thecouch and fell asleep for the afternoon.She awoke in time to make her children dinner. After feeding them and putting themto bed she had two glasses of wine while watching a movie. She felt tipsy, since shehad not eaten anything all day. Feeling exhausted, at 9 pm she went to bed for thenight. Which of the following items would be a way to manipulate the cash flow from operating activities amount on the statement of cash flows? a. Adding depreciation back to net income to determine cash flow from operating activities. b. Including interest expense and tax expense in the calculation of cash flow from operating activities. c. Recording an item that should be recorded as an operating activity as an investing activity. d. The cash flow statement cannot be manipulated. Which relation is a function? Explain your reasoning. Match each explanation to the appropriate blanks in the sentences on the right. 1. the atomic radius decreases 2. the number of gas molecules decreases 3. molar mass and structure complexity decreases 4. structure complexity decreases 5. molar mass decreases 6. each phase (gas, liquid, solid) becomes more ordered A (I_2(g), Br_2 (g), Cl_2 (g), F_2 (B): The ranking can best be explained by the trend entropy decreases as______. B (H_2O_2 (g), H_2S(g), H_2O(g): The ranking can best be explained by the decreases a trend entropy decreases as_______. C. (C(s, amorphous), C(s, graphite), C(s, diamond): The ranking can best be explained by the trend entropy decreases as_______. 4x + 5y = 19 , 5y - 4x = 38